1000 CR - Test 2 Question 18

This topic has expert replies
Senior | Next Rank: 100 Posts
Posts: 91
Joined: Tue May 01, 2007 4:04 pm

by Nisha1218 » Thu Nov 01, 2007 5:44 pm
After reading the question and stimulus, we are looking for an answer that would strengthen the reason that if Lark employees join the program, the will later feel work dissatisfaction. The reason that the answer is not B is because the same dissatisfaction was felt by the employees not even enrolled in the program - this actually weakens the reason that the program is the cause for dissatisfaction.

E is the only answer that strengthens the conclusion.

Senior | Next Rank: 100 Posts
Posts: 43
Joined: Thu Sep 11, 2008 10:05 pm
Location: India

by poonam197 » Sun Dec 14, 2008 4:42 am
IMO, the answer should be A.

Please someone explain clearly the correct is E and not A.

Legendary Member
Posts: 541
Joined: Thu May 31, 2007 6:44 pm
Location: UK
Thanked: 21 times
Followed by:3 members
GMAT Score:680

by rohangupta83 » Sun Dec 14, 2008 10:20 am
poonam197 wrote:IMO, the answer should be A.

Please someone explain clearly the correct is E and not A.
Here is an explanation - hopefully its clear. --- imo E

Ok, I'll break the approach to this question in steps.

Step 1 - identify the conclusion:

Obviously, any workers who participate in a Quality Circles program will, as a result, become less satisfied with their jobs.

Step 2 - What is the question asking us to do?

weaken the conclusion drawn above EXCEPT

Basically, we have to cross out all statements which weaken the conclusion.

THAT IS:

Cross out all choices which provide an alternative reason for the workers/operators who participated in the training to be dissatisfied.

Cross out all choices which provide a reason for higher satisfaction after training.

Step 3 - Look at answer choices

Choice A - The second survey occured during recession so people are bound to feel dissatisfied - OUT

Choice B - Provides reasoning that training improved satisfaction - opposite of our conclusion

Choice C - Provides reasoning that training improved satisfaction - opposite of our conclusion

Choice D - Same as C

Choice E - Neither weakens nor strengthens. Hence this is IT! (Out of Scope)

Remember the question is asking you to select the choice which simply does not weaken the answer choice. It does not ask you to select an answer choice that strengthens the argument.

Senior | Next Rank: 100 Posts
Posts: 58
Joined: Sun Jun 17, 2007 8:24 am

by vish150783 » Mon Dec 15, 2008 6:45 am
Are these 1000 CR from reliable sources? I read in the forum somewhere its not recommended to work with 1000 CR and 1000 SC.

They could screw up your logic quite a bit.

Legendary Member
Posts: 708
Joined: Sun Jun 01, 2008 4:59 am
Location: USA
Thanked: 13 times
Followed by:1 members

by niraj_a » Mon Dec 15, 2008 7:28 am
E looks good.

the conclusion is - anyone who participates in QC will become less satisfied with his/her job.

the correct answer choice will be irrelevant or will not weaken this conclusion.

B tricks you by being wordy. B implies that participation in QC is immaterial to employees becoming less satisfied i.e. employees could become less satisfied even if the program wasn't there. this weakens the conclusion above.

A, C, D also weaken. so by POE, only choice E remains. E doesn't really have any effect on the conclusion. what the employees expected going into QC doesn't matter in forming the conclusion above.

Senior | Next Rank: 100 Posts
Posts: 43
Joined: Tue Mar 31, 2009 9:54 am
Thanked: 2 times

by glorydefined » Tue Sep 08, 2009 6:31 am
excellent explanation by rohan :)

Senior | Next Rank: 100 Posts
Posts: 79
Joined: Sun Oct 02, 2016 2:08 am
Thanked: 1 times

by TheGraduate » Fri Oct 07, 2016 12:22 am
I selected option E. However on closer examination a lot of questions crept into my mind.

Stimulus: Independent Surveys (plural) of employee attitudes: (employees less satisfied with work after 2 yrs.) correlated to (employees participating in Quality Circles program)

Concl: (participating in Quality Circles program) ----> (decline in satisfaction) [causality implied]

A) The second survey occurred during a period of recession when rumors of cutbacks and layoffs at Lark Manufacturing were plentiful ---- Seems to imply that cutbacks and layoffs were responsible (alternate causality) but, then again, speaks specifically about the second survey. What about the other surveys? Is one survey among many credible enough to weaken the conclusion?

C) While participating in Quality Circles at Lark Manufacturing, machine operators exhibited two of the primary indicators of improved job satisfaction: increased productivity and decreased absenteeism
---- Seems to weaken since it goes contrary to the decline in satisfaction noted in the simulus ans conclusion. However, merely finding INDICATORS of improved satisfaction may not actually indicate job satisfaction. Increased productivity and decreased absenteeism may be caused by other factors than improved satisfaction.

eg. +satisfaction --> Increased productivity
or, +satisfaction --> decreased absenteeism
but +satisfaction may not be the only cause.


OR, Increased productivity U decreased absenteeism --> +satisfaction ?
( U being logical or)


In other words how should I interpret "indicator"?

D) Several workers at Lark Manufacturing who had participated in Quality Circles while employed at other companies reported that while participating in Quality Circles in pervious companies, their work satisfaction has increased. ---- Quality Circles in other companies do not matter. Or, is the clause "any workers who participate in a Quality Circles program will, as a result, become less satisfied with their job" the real reason behind (D) being a weakener.

E)The machine operators who participate in Quality Circles reported that when the program started, they felt that participation might improve their work situation ---- IS this a strengthener or simply irrelevant?

The main question is how much do we need to read into each option exactly?

Secondly, how close is this to a real GMAT question ?

Please advice. Any insight would be greatly appreciated.